Statica vraag

Moderators: jkien, Xilvo

Reageer
Berichten: 4

Statica vraag

[attachment=4701:statica_som_6_78.jpg]

goedenavond dames en heren

op school is ons gevraagd naar deze som te kijken en alvast te bestuderen maar ik kom er niet goed uit.

omdat als ik hier een VLS (vrijlichaamschema) van maak nogal veel onbekenden heb.

zoals op de foto te zien is zit er een staaf tussen B en E ,en E en D dit zijn volgens mij 2 pendelstaven?

ik kom uit op 10 onbekenden en weet hier geen oplossing voor..

iemand een idee over een goede aanpak? ik hoop dat de afbeelding en de vraag te zien is.

mvg
Bijlagen
vraag_6_78.jpg
vraag_6_78.jpg (30.2 KiB) 477 keer bekeken

Berichten: 4

Re: Statica vraag

vraag_6_78.jpg
vraag_6_78.jpg (30.2 KiB) 479 keer bekeken
sorry er was iets foutgegaan met de foto

hieronder volgt nog mijn vls

mvg
vls6_78.jpg
vls6_78.jpg (39.19 KiB) 479 keer bekeken
hierbij vls van de opgave i.p.v 10 onbekenden trouwens 8 want dx en bx zijn er dus 2 keer (actie min reactie?)

ik denk zelf dat ik de opgave en dus ook het vls op moet splitsen of ergens bij een scharnier punt moet losmaken

maar toch lukt het niet helemaal

mvg

Berichten: 150

Re: Statica vraag

Waarom zie jij F als een scharnier? Dit is overduidelijk een roloplegging, bijgevolg valt jouw kracht Fx weg, zodat je maar 9 onbekenden overhoudt.

Maak van elk van je staven een vrijgemaakt lichaam, dan kan je 3 keer 3 vergelijkingen opstellen.

9 onbekenden, 9 vergelijkingen, systeem opgelost...

Berichten: 4

Re: Statica vraag

je hebt gelijk foutje :eusa_whistle:

maar dan nog iets haha

ik heb nu 8 vergelijkingen opgesteld

maar van de horizontale balk weet je de afstand tussen B en E niet

ik heb hier dus maar om punt E gedraait maar ik weet niet of het goed is.

ik kom ook niet op de goede antwoorden uit via de vergelijkingen

ik zal mijn 9 vergelijkingen inscannen met vls (zie hieronder)

in ieder geval bedankt al voor je reactie

mvg

[attachment=4707:3vergeli...gen1balk.jpg]

[attachment=4708:3vergeli...gen2balk.jpg]

[attachment=4709:3vergeli...gen3balk.jpg]
Bijlagen
3vergelijkingen3balk.jpg
3vergelijkingen3balk.jpg (38.08 KiB) 479 keer bekeken
3vergelijkingen2balk.jpg
3vergelijkingen2balk.jpg (37.25 KiB) 479 keer bekeken
3vergelijkingen1balk.jpg
3vergelijkingen1balk.jpg (29.87 KiB) 485 keer bekeken

Berichten: 4.502

Re: Statica vraag

Volgens meetkundige wetten is BE de helft van AF !

En de rol heeft met heel veel turen een straal van 1 mtr!

Reageer